Saltar al contenido

¿Las funciones continuas son diferenciables en un conjunto medible?

Nuestros mejores programadores agotaron sus depósitos de café, por su búsqueda todo el tiempo por la resolución, hasta que Alfredo halló el resultado en Gogs y ahora la comparte contigo.

Aquí hay un intento de salvar la solución de Matthew Pancia, que desafortunadamente dependía de un incontable unión sobre todas las posibles derivadas.

Dado $ f $ podemos definir, de la manera obvia, una función continua $ F: mathbb R times ( mathbb R setminus 0) to mathbb R $ tal que $ f $ sea diferenciable en $ x $ exactamente cuando $ lim_ h to 0 F (x, h) $ existe. La formalización habitual de esto es $$ existe y: forall varepsilon: existe delta: forall h: | h | < delta Rightarrow | F (x, h) -y | < varepsilon $$ Clásicamente todas las variables aquí son reales, pero es fácil ver que podemos restringir $ varepsilon $ y $ delta $ a $ mathbb Q $ sin cambiar el significado. También podemos restringir $ h $ a $ mathbb Q $ porque $ F $ es continuo. Sin embargo, es esencial que $ y $ pueda ser un real arbitrario, porque de lo contrario estaríamos buscando puntos donde $ f $ sea diferenciable con derivada racional, que es algo bastante diferente.

Sin embargo, también podemos formalizar la existencia de un límite como $$ forall varepsilon: existe delta: existe Y: forall h: | h | < delta Rightarrow | F (x, h) -Y | < varepsilon $$ Esto funciona porque $ mathbb R $ está completo; es esencialmente lo mismo que cambiar "tiene un límite" sobre un secuencia a “es Cauchy”. Los argumentos de que $ varepsilon $, $ delta $ y $ h $ pueden restringirse a los racionales funcionan como antes, pero ahora $ Y $ también se puede tomar como racional en cada caso.

Para cada elección particular de $ varepsilon $, $ delta $, $ Y $ y $ h $, el conjunto de $ x $ tal que $ | h | < delta Rightarrow | F (x, h) -Y | < varepsilon $ está abierto y, por lo tanto, Borel.

Ahora maneje cada uno de los cuantificadores de adentro hacia afuera: para cada elección de $ varepsilon $, $ delta $ y $ Y $, el conjunto de $ x $ tal que $$ forall h: | h | < delta Rightarrow | F (x, h) -Y | < varepsilon $$ es una intersección contable de conjuntos de Borel y, por lo tanto, Borel. Para cada elección de $ varepsilon $ y $ delta $ el conjunto de $ x $ tal que $$ existe Y: forall h: | h | < delta Rightarrow | F (x, h) -Y | < varepsilon $$ es una unión contable de conjuntos Borel y, por tanto, Borel. Etcétera. En la parte superior encontramos que el conjunto de puntos de diferenciabilidad es Borel y, por tanto, en particular medible.

Teorema 1
Sea $ f $ una función medible en $ (a, b) $. Entonces la función $$ g_n (x) = sup left f (x + h): h in (0, frac 1 n) cap (0, bx) right $ $ es medible.

Prueba
Arregle $ c in mathbb R $. Queremos demostrar que el conjunto $ A = x in (a, b): g_n (x)> c $ es medible, entonces $ g_n $ será medible. Sea $ x_0 en A $, entonces existe $ h_0 in (0, frac 1 n) cap (0, b-x_0) $ tal que $ f (x_0 + h_0)> c $. Defina $$ delta_1 = frac 1 2 left ( min left ( frac 1 n, b-x_0 right) -h_0 right) qquad delta_2 = frac 1 2 left ( min left ( frac 1 n, b-x_0 right) + h_0 right) $$ luego para todos los $ x in (x_0- delta_1, x_0 + delta_2) $ tenemos $ x_0 + h_0 in x + h: h in (0, frac 1 n) cap (0, bx) $. Como consecuencia $ g_n (x) geq f (x_0 + h_0)> c $, entonces $ x in A $ para todos $ x in (x_0- delta_1, x_0 + delta_2) $. Esto significa que $ A $ está abierto, por lo tanto medible.

Teorema 2
Sea $ f $ una función medible en $ (a, b) $, entonces el conjunto $$ A = x in (a, b): exist f ‘(x) in mathbb R $$ es medible.

Prueba
Extienda $ f $ por igualdades: $ f (x) = f (a) $ por $ xb $. Para cada $ c in mathbb R $ considere la función medible $ phi (x) = f (x) -cx $. Del teorema 1 se deduce que $$ psi_n (x) = sup left phi (x + h): h in (0, frac 1 n) cap (0, bx) right $$ es medible. Entonces tenemos la función medible $$ psi_n (x) – phi (x) = sup left phi (x + h) – phi (x): h in (0, frac 1 n) cap (0, bx) right . $$ Por tanto, el conjunto $$ B = x in (a, b): psi_n (x) – phi (x)> 0 = $$ $$ left x in (a, b): sup left frac f (x + h) -f (x) h: h in (0, frac 1 n) cap (0, bx) right > c right $$ es medible. Como $ c in mathbb R $ es arbitrario, entonces las funciones $$ f_n (x) = sup left frac f (x + h) -f (x) h: h en (0, frac 1 n) cap (0, bx) right $$ son medibles. Entonces concluimos que la función $$ overline f ‘_ + (x) = overline lim limits_ h to 0 + frac f (x + h) -f (x) h = lim limits_ n to infty f_n (x) $$ también es medible. De manera similar, podemos probar que las funciones $$ underline f ‘_ + (x) = underline lim limits_ h to 0 + frac f (x + h) -f (x) h = lim limits_ n to infty f_n (x) $$ $$ overline f ‘_- (x) = overline lim limits_ h to 0- frac f (x + h) -f (x) h = lim limits_ n to infty f_n (x) $$ $$ underline f ‘_- (x) = underline lim limits_ h to 0 – frac f (x + h) -f (x) h = lim limits_ n to infty f_n (x) $$ son mensurables. Finalmente, el conjunto $ A = x in (a, b): exist f ‘(x) in mathbb R $ es medible porque $$ A = x in (a, b): overline f ‘_ + (x) = underline f’ _ + (x) = overline f ‘_- (x) = underline f’ _- ​​(x) in mathbb R $$

Lo que sigue está tomado de una publicación mía en ciencia matemática del 16 de mayo de 2006.

Sea $ f: mathbb R rightarrow mathbb R $ arbitrario. Entonces, el conjunto de puntos donde $ f $ no tiene una derivada finita es un conjunto $ G _ delta sigma $. Esto es también true para el conjunto de puntos donde $ f $ no tiene una derivada izquierda finita o el conjunto de puntos donde $ f $ no tiene una derivada derecha finita. Una prueba de esto se da después del siguiente párrafo.

Por el contrario, dado cualquier $ G _ delta $ set $ G_1 $ y $ G _ delta sigma $ set $ G_2 $ tal que $ G_2 $ tiene la medida de Lebesgue cero, entonces existe un continuo function $ f: mathbb R rightarrow mathbb R $ cuyo conjunto de no diferenciabilidad es precisamente $ G_1 cup G_2. $ Esto fue probado por Zygmunt Zahorski (1914-1998) en 1941 y reeditado en francés en 1946 (Zbl 61,11302; MR 9.231a). Para el periódico francés de 1946, consulte http://tinyurl.com/3zwca22.

La primera declaración anterior para continuo $ f $ se sigue del hecho de que $ f $ tiene una derivada finita en $ x $ si y solo si $ x $ pertenece a

$$ bigcap_ k = 1 ^ infty ; bigcup_ n = 1 ^ infty ; ; bigcap_ eta ; ; bigcap_ < frac 1 n ; left x: ; ; left $$

Para ver que este resultado es válido para una función arbitraria $ f $ (no necesariamente continua), relativizamos la descomposición anterior a $ C (f) $, el conjunto de continuidad de $ f $. Es decir, considere los conjuntos $ leq frac 1 k . $ Esto mostrará que el conjunto donde $ f $ tiene una derivada finita es un $ F _ sigma delta $ subconjunto del espacio $ C (f), $ donde $ C (f) $ tiene la topología subespacial de $ mathbb R. $ (“Subconjunto”, porque cada punto en el que existe una derivada finita es un punto de continuidad). Por lo tanto, este conjunto es la intersección de $ C (f) $ con un $ F _ sigma delta $ establecido en $ mathbb R $ (cosas de topología del subespacio), y por lo tanto este conjunto es $ F _ sigma delta $ en $ mathbb R $ (porque $ C (f) $ tiene una clase Borel más baja, siendo $ G _ delta $ en $ mathbb R). $

valoraciones y reseñas

Tienes la opción de añadir valor a nuestro contenido informacional aportando tu veteranía en las aclaraciones.

¡Haz clic para puntuar esta entrada!
(Votos: 0 Promedio: 0)


Tags :

Utiliza Nuestro Buscador

Deja una respuesta

Tu dirección de correo electrónico no será publicada. Los campos obligatorios están marcados con *